10
$\begingroup$

An 8x8 Sudoku where you technically only have to figure out 32 cells! Twice Half the cells, double the clues!

Penpa Link

enter image description here

Rules

  • Normal Sudoku rules apply. Values are 1-8. Boxes are the 4x2 regions
  • Cells that are a chess knight's move apart must not contain the same number
  • Digits along a thermometer must strictly increase, starting from the bulb end
  • Numbers that appear in the cells between the black dot must have a ratio of 2
  • "Diagonally opposite" cells must contain the same number.
    • As an expression: digit(row, column) = digit(9 - row, 9 - column), where the row and column values are 1-indexed.
    • As an example: R1C1 must be equal to R8C8. R2C4 must be equal to R7C5, and so on.
$\endgroup$
3
  • $\begingroup$ Would "180 degree rotationally symmetric" be an accurate descriptor of your "Diagonally opposite" expression? $\endgroup$
    – bobble
    Apr 10, 2021 at 21:47
  • $\begingroup$ @bobble _ Yes. _ $\endgroup$ Apr 10, 2021 at 21:52
  • $\begingroup$ I think for the benefit of the solver it would be good to define what a "knight's move" is $\endgroup$
    – HTM
    Apr 10, 2021 at 23:29

1 Answer 1

5
$\begingroup$

If I've interpreted the rules correctly, then this should be the solution:

clones_sudoku_sol

Explanation

We start off by

copying over the given digits to their rotationally symmetric cell, as well as copying the thermometers for ease of solving: clones_sudoku_1

Next, we

can place the 1s. Using the knight move rule, we can deduce that R4C3 must be a 1. Now, either R1C2 or R1C4 is a 1, but if we make R1C2 a 1, we find that R2C5 would have to be a 1 as well, which is not possible since a 1 cannot occur in the middle of a thermometer. Thus, R1C4 is a 1, which then forces R2C7 to be a 1: clones_sudoku_2

Now, we can note that

R5C3 has to be at least 5, since the start of the thermometer it is a part of has to be at least 2. As the ratio between R5C2 and R5C3 has to be 2, we conclude that R5C3 is either 6 or 8, and R5C2 is either 3 or 4. For ease of explanation, we'll call a cell with either 6 or 8 "blue" and a cell with either 3 or 4 "green." By the knight's move rule, we can deduce, in order, than R3C1 is blue; then, R3C4 is green; and finally, R3C3 is a 5: clones_sudoku_3

Building off of this, we can now determine

the location of the remainder of the 5s. We can see that either R2C2 or R2C4 is a 5, but R2C2 being a 5 would violate the thermometer rule, so R2C4 has to be the 5, which forces R1C7 as a 5. This in turn means that R8C4 has to be blue (since the other candidate R8C2 is taken by a 5), so R7C7 has to be blue as well. Finally, we determine that due to the thermometer, R4C5 is either a 2 or a 3, and so is R3C2 - we will call that condition "red" from now on. clones_sudoku_4

The next step involves a bit of "what if?" reasoning - if there's a more straightforward logical way to proceed, then let me know:

Notice that R8C1 and R8C3 are green and red, in some order. In particular, R8C3 is one of 2, 3, or 4. Now, suppose that R6C3 is a 6. This would force R8C3 to be either a 2 or a 3, since there are two numbers in between the ends of that thermometer - hence, it is red. This means R8C1 is green; in fact, R8C1 would be a 3, since R5C2 has to be a 3 by our ratio condition. Thus, the numbers in between R8C3 and 6 on that thermometer have to be 4 and 5. However, we cannot use a 5 here as we've already placed one in each row, column, and box. This gives us a contradiction, and since the only assumption we made here was that R6C3 was a 6, we conclude that R6C3 has to be an 8 instead. This means that R6C2 is a 4, making all blue cells 8 and all green cells 4: clones_sudoku_5

We're at the home stretch now! Next, we see that

R8C3 has to be a 2 - if it were a 4, then the thermometer containing it would have to go "4, 6, 7, 8," and the thermometer overlapping it cannot be completed at the end; if it were a 3, then R8C1 would be a 4, and again the thermometer would have to go "3, 6, 7, 8." Thus, R8C3 is a 2, R8C1 is a 4, and all red cells are 2s. We can fill out the grid using these deductions: clones_sudoku_6

And now, we can finish it off pretty simply:

R7C4, R6C4, and R6C5 have to be 3, 6, and 7 respectively. This forces R2C8 to be a 6 and R3C7 to be a 7, completing that thermometer. The rest of the grid can be completed straightforwardly, using sudoku and knight move rules as necessary to get our final solution: clones_sudoku_sol

$\endgroup$
1
  • 1
    $\begingroup$ That's the correct solution, well done! The "What if"s were intended, I hope they didn't feel unfair in this case. $\endgroup$ Apr 11, 2021 at 7:29

Your Answer

By clicking “Post Your Answer”, you agree to our terms of service and acknowledge you have read our privacy policy.

Not the answer you're looking for? Browse other questions tagged or ask your own question.